Improper Integral Question (exponential integral)

calculusimproper-integrals

I'm trying to prove that
$\int_{0}^{\infty} x^{t} e^{-x} dx$ diverges for $t \leq -1.$

I figured I'd split it up into
$\int_{0}^{1} x^{t} e^{-x} dx + \int_{1}^{\infty} x^{t} e^{-x} dx.$

If I can show that the first integral in the sum diverges, then I think I know the original integral diverges.

But how should I go about showing that the first integral diverges?
In an earlier problem, I showed that $\int_{0}^{1} x^t dt$ diverges for $t \leq -1$. I was thinking of trying to find an integral that is less than my original integral but also diverges, but I got stuck.

Any ideas?

Best Answer

HINT:

Note that $ x^t e^{-x}\ge e^{-1} x^t$ for $x\in [0,1]$.

Related Question